LSAT and Law School Admissions Forum

Get expert LSAT preparation and law school admissions advice from PowerScore Test Preparation.

 Nikki Siclunov
PowerScore Staff
  • PowerScore Staff
  • Posts: 1362
  • Joined: Aug 02, 2011
|
#6223
Argumentation is not an exact science, and I wouldn't try to make it into one. Let's just think the argument through. The author concludes that people's fears of a nuclear accident are justified, which clearly indicates a presumption of some real danger. We cannot know exactly how "unsafe" the nuclear plant needs to be for such fears to be justified. But there is a presumption of a real danger: if there weren't, the conclusion would make no sense. This assumption, in turn, is motivated by the sole observation that the government took certain steps to insure the industry against potential harm.

Based on the information contained in the premises, however, we have no evidence that real danger actually exists. This is the central flaw this argument. The government's actions do not indicate "real danger" that justifies fear, just like your getting a tetanus shot does not mean you face a real danger of becoming infected. While it is possible that you got the shot because you fear infection, your actions do not prove that you had such fears, let alone suggest that your fears were justified. You may have gotten a tetanus shot just because your doctor told you it's a wise thing to do.

Hope this clears things up!
 voodoochild
  • Posts: 185
  • Joined: Apr 25, 2012
|
#9347
Experts,
On this one, I was down to a and b. I ended up choosing A because I thought I know government's position on the safety of nuclear plants but not on nuclear plants. (One reason why A) wins)

(Second reason why A) wins) Also, the question says..."if all statements offered in support of editorial's conclusion are correct...." Hence, I thought that the government's claim about safety is wrong and the public's fear on nuclear accidents is indeed correct.

Not sure. Please help.

Thank you
 BethRibet
PowerScore Staff
  • PowerScore Staff
  • Posts: 200
  • Joined: Oct 17, 2012
|
#9352
Hi Voodoochild,

What we know from the question stem is that each statement correctly describes one of the government's "positions", meaning their claims or stances. We have no way to know whether any of the statements are actually true, only that they accurately identify what the government states.

Being clear on that helps to rule out answer choice A, because it definitively tells us that one of the claims is false, and we have no way to know that.

What we do know is that the government claims the plants are safe (from nuclear accident), but also claims that the nuclear industry needs to be protected from an outcome which could only occur if there were a nuclear accident. Though both claims are the government's positions, they can't both be correct claims. If the plants are safe, then the industry does not need protection from an event that could only occur if they are unsafe. They may or may not be safe -- we don't know *which* claim is incorrect, which again, is why we rule out A. B essentially states that the claims are inconsistent, which must be true given the incompatibility of the government's two positions.

Hope this helps!
Beth
 SherryZ
  • Posts: 124
  • Joined: Oct 06, 2013
|
#12171
Hi! Thank you for your generous help!

Oct 1999 LSAT, Sec 1, LR Q21:

I was distracted by A and B. Eventually I chose A but the correct answer is B.

Could you tell me why A is wrong and why B is right???? I am so confused.


---Sherry
 Nikki Siclunov
PowerScore Staff
  • PowerScore Staff
  • Posts: 1362
  • Joined: Aug 02, 2011
|
#12197
Hi Sherry,

This is a tricky one. Let's take a look at the editorial's argument more closely:
  • The government claims that we are safe from nuclear accidents, but wants to limit the nuclear industry's financial liability in case of an accident. Liability, however, is only a threat if someone is injured in a nuclear accident. So, nuclear plants are dangerous (and our fear is justified).
The conclusion clearly implies that nuclear plants are dangerous, contrary to what the government might claim. In other words, yes - the editorial concludes that the government's claim about the safety of nuclear plants is false. So, what's wrong with answer choice (A)?

The key is in the question stem. They didn't ask you to identify the main point of the argument, or even assume that the argument is true in its entirety. You are only instructed to take as true the statements offered in support of the editorial's conclusion, not the conclusion itself. Basically, if all the premises of the editorial's argument are true, what else must be true on the basis of those statements?

The editorial's premises, by themselves, do not establish that nuclear plants are actually dangerous. The editorial's argument is actually quite weak: just because the government wants to limit liability in case of an accident doesn't mean that there is actually any danger of a nuclear accident. We can't prove that the government is actually lying about the safety of nuclear power plants, because we cannot draw any factual conclusions about whether or not nuclear power plants are safe. There is a difference between fact and opinion, and that difference is crucial on the LSAT! The only thing we can conclude is that the government's position is inconsistent: they say one thing (we are safe), but do another (protect the industry against liability in case of an accident). This behavior may suggest that we aren't as safe as the government claims, but we cannot prove risk as a matter of fact. All we can prove is inconsistency of the government's position, which is answer choice (B).

Hope this helps! :-)
 PB410
  • Posts: 39
  • Joined: Apr 01, 2017
|
#39032
I'm having difficulty understanding this stimulus and why answer choice D would be wrong. If the government claims in the first sentence that power plants are entirely safe, and then later in the stimulus we a presented with a conditional chain of

unlimited liability poses a threat > injury claims sustained against industry > government admits such claims to be sustained > injury > result from nuclear accident.

But if we are told in the beginning that power plants are safe wouldn't that trigger a contrapositive chain that negates injury from nuclear threat and ends at unlimited liability poses no threat? Answer choice D states, that unlimited financial liability poses not threat.
 AthenaDalton
PowerScore Staff
  • PowerScore Staff
  • Posts: 296
  • Joined: May 02, 2017
|
#39358
Hi PB410,

There are two contradictory positions the government is taking in this problem:

First, that nuclear plants do not pose any threat of injury to the public.

Second, that nuclear plants need to have their liability for personal injury claims capped because the "unlimited liability" faced by nuclear plants would ruin the industry.

The problem is that if nuclear plants truly are 100% safe, as the government contends, then they wouldn't need any limitations on how much liability they would face for personal injuries. The only reason a nuclear plant would be successfully sued for a personal injury is if the plant was not, in fact, 100% safe. So the government's two positions are incompatible with each other.

We are told to take all of the statements describing the government's position as an actual reflection of the government's stance. The government cannot both claim that caps on liability are necessary and that a nuclear plant will never cause an accident. If it's really the case that a nuclear plant will never cause an accident, then the liability caps would not be justified.

I hope this helps clarify things for you. Good luck studying!

Athena Dalton
 Leela
  • Posts: 63
  • Joined: Apr 13, 2019
|
#64458
I think I am repetitively misreading the stimulus here. Is the problem with the argument that the public is concerned with physical safety, but the government is drawing a conclusion based on financial safety? Could someone please walk me through the premises and the conclusion?
User avatar
 Dave Killoran
PowerScore Staff
  • PowerScore Staff
  • Posts: 5852
  • Joined: Mar 25, 2011
|
#64478
Leela wrote:I think I am repetitively misreading the stimulus here. Is the problem with the argument that the public is concerned with physical safety, but the government is drawing a conclusion based on financial safety? Could someone please walk me through the premises and the conclusion?
Hi Leela,

The stimulus is written from the perspective of an editorial, so you should expect a comment of some sort on a situation, which is exactly what we get, and the topic is what the government is saying and whether the public's concerns there are reasonable.

So, in very rough form, here's what we have: the government says that people shouldn't worry about a nuclear accident. But the editorial points out the government recently made a move to limit liability in case of an nuclear accident, and why the heck would they do that unless there was real concern over an accident? Since that action seems to contradict their words, the conclusion is that the public indeed should be worried about a nuclear accident.

Answer (B) plays with that "actions speak louder than words" issue on the part of the government, and in typical LSAT fashion, describes it in logical terms as an inconsistency.

Please let me know if that helps. Thanks!
 bonnie_a
  • Posts: 32
  • Joined: Jun 05, 2021
|
#89458
Why is C incorrect? I first picked this answer because I thought the government was unsuccessful in elaborating on why they want to limit the liability of nuclear power plants. In other words, with their reasons being incoherent altogether, they "misrepresented" their reasons for protecting it. Also, I did not consider B because it wasn't that "the government's position" on nuclear power plants was inconsistent. Their thoughts/stance on them may have remained the same while the reasons they provided made it seem as if they were not so. Thank you!

Get the most out of your LSAT Prep Plus subscription.

Analyze and track your performance with our Testing and Analytics Package.